본문 바로가기
수리통계를 위한 해석학

찍먹 측도론 3 Dynkin $\pi - \lambda$ theorem(Dynkin Pi-lambda theorem)

by juhongyee 2024. 7. 11.

 

 

서론

지난 시간에는 Carathéodory's extension theorem에 대해 알아보았습니다. 그런데 이 정리에서 $\sigma$-finite라는 조건을 추가했을 때는 그 measure가 unique 하게 존재합니다.

이는 Probability measure에서 일반적인 상황입니다. 그렇다면 한 번 증명해봐야겠죠?

 

이 uniqueness를 증명하기 위한 초석은 Dynkin $\pi - \lambda$ theorem을 증명하는 것입니다.

이를 위해 $\pi$ system과 $\lambda$ system에 대해서도 차근차근 알아보죠!

 

본론

Definition of $\pi$ system

$\pi$ system의 정의부터 찬찬히 알아봅시다. 다음은 그 서술입니다.


A collection of $\mathcal{A}$ is a $\pi$ - system if $\forall A,B \in \mathcal{A}$ then $A \cap B \in \mathcal{A}$ and $\varnothing \in \mathcal{A}$


간단하죠? 교집합과 공집합이 $\mathcal{A}$에 속하면 $\pi$ system입니다. ring의 하위호환으로 union은 성립하지 않아도 됩니다.

 


Definition of $\lambda$ system

이번엔 $\lambda$ system의 정의입니다.


A collection of subsets $\mathcal{L}$ is a $\lambda$ system if

 

- $\Omega \in \mathcal{L}$

- $A,B \in \mathcal{L}$ s.t if $A \subset B$ then $B-A \in \mathcal{L}$

- if $\{A_i\}_{i=1}^{\infty}$ pairwise disjoint, $A_i \in \mathcal{L}$, the $\bigcup_{i=1}^{\infty} A_i \in \mathcal{L}$


 

굉장히 $\sigma$ - field와 비슷하죠? 하지만 다른 점이 있습니다. 두 번째를 자세히 보시면 차집합에 대해 닫혀있는 게 아닙니다. 부분집합인 경우 차집합이 $\mathcal{L}$에 속하는 거죠.

또 세 번째의 pairewise 부분에서도 겹치지 않을 때의 countable union만 $\lambda$ system에 속합니다.

 

이 지식들을 토대로 Dynkin $\pi - \lambda$ theorem을 증명해 봅시다.

 

Dynkin $\pi - \lambda$ theorem

일단 이 정리가 무엇인지 알아야겠죠.


Let $\mathcal{A}$ be a $\pi$ system, $\mathcal{L}$ be a $\lambda$ system.

And $\mathcal{A} \subseteq \mathcal{L}$. Then $\sigma (\mathcal{A}) \subseteq \mathcal{L}$


$\mathcal{A}$가 $\pi$ system이며 $\lambda$ system에 속한다면 그 generated $\sigma$-field도 $\mathcal{L}$에 속한다는 의미입니다. 원래 $\sigma$-field는 유일하지 않지만 가장 작은 $\sigma$-field는 유일합니다. 가장 작다(smallest)의 정의는 아래에서 보이겠습니다.

 

자 증명해 봅시다.

proof of Dynkin $\pi - \lambda$ theorem

먼저 $\mathcal{L_0}$가 smallest $\lambda$ system s.t $\mathcal{A} \subset \mathcal{L_0}$라고 해봅시다.

 

smallest?

그러면 여기서 $\mathcal{A}$를 포함하는 smallest의 정의를 알 필요가 있습니다.

이의 정의는 $\mathcal{A}$를 포함하는 모든 $\lambda$ system의 교집합입니다. 그러면 다시 질문이, $\lambda$ system의 교집합은 교집합인지 생각해 봐야겠죠.

$\mathcal{L_0} = \bigcap_{\mathcal{L}} \mathcal{L}$이 $\lambda$ system인지 알아봅시다. $\lambda$ system의 정의를 따라가면 됩니다.

 

- 일단 $\Omega \in \mathcal{A}$는 자명합니다. 모든 $\mathcal{L}$이 $\Omega$를 가지고 있기 때문입니다.

- 다음은 차집합인데, $A,B$가 $\mathcal{L_0}$에 속한다고 해보죠. 그러면 모든 $\mathcal{L}$에 $A,B$가 속해있었다는 뜻입니다. 그러면 모든 $\mathcal{L}$들은 $\lambda$ system의 정의에 의해 $A-B$도 가지고 있었습니다. 그러므로 그들의 교집합인 $\mathcal{L_0}$도 $A-B$를 가집니다.

 

- countable union도 위와 정확히 동일한 원리로 성립합니다.

 

즉, $\mathcal{L_0}$가 잘 존재하여 우리가 선택할 수 있습니다.

($\mathcal{L}$ s.t $\mathcal{A} \subset \mathcal{L}$의 존재성에 대해 더 고려할 점이 있지만 별 거 아니므로 패스하겠습니다.)

 

다시 본 증명으로 돌아옵시다.

 

우리의 목표는 $\mathcal{L_0}$가 $\pi$ system이라는 것과, $\pi$ system과 $\lambda$ system을 동시에 만족하는 set의 집합은 $\sigma$ field임을 보이는 것입니다.

 

결국 $\sigma (\mathcal{A}) \subseteq \mathcal{L_0} \subseteq \mathcal{L}$을 보이고자 합니다.

 

step1 Show that $\mathcal{L_0}$ is contains all intersections with element of $\mathcal{A}$

먼저 $\mathcal{L'} = \{B \in \mathcal{L_0} : B \cap A \in \mathcal{L_0}, \forall A \in \mathcal{A}\}$ 을 정의합시다.

 

먼저 $\mathcal{L_0}$는 $\mathcal{A}$를 포함한다는 걸 기억합시다. 그러면 $\mathcal{L'}$은 최소 $\mathcal{A}$는 포함하겠죠. 왜냐하면 $A \in \mathcal{A}$들은 $\mathcal{A}$가 $\pi$ system이므로 정의를 만족합니다.

 

즉, $$\mathcal{A} \subset \mathcal{L'}$$입니다.

 

자, 이제 $\mathcal{L'}$이 $\lambda$ system임을 보여봅시다.

$\lambda$ system의 정의를 하나하나 체크합시다.

 

- 일단 $\Omega$를 당연히 포함합니다. $\mathcal{A}$를 부분집합으로 가지고 있기 때문입니다.

 

- $B_1,B_2 \in \mathcal{L'}$ 이고 $B1 \subset B2$라고 합시다. 그러면 $\mathcal{L'}$의 정의에 의해 $B_1 \cap A, B_2 \cap A \in \mathcal{L_0}$입니다.

그렇다면 $(B_1 \cap A) - (B_2 \cap A) = (B_2 - B_1) \cap A \in \mathcal{L_0}$가 만족함을 쉽게 보일 수 있습니다. $B_1$이 $B_2$의 부분집합이고, 각각이 $\mathcal{L_0}$에 속하므로 그 차도 $\mathcal{L_0}$에 속합니다..

 

즉, $$\therefore B_2-B_1 \in \mathcal{L'}$$입니다.

 

- $\{B_i\}_{i=1}^{\infty} \in \mathcal{L'}$ 이고 pairwise disjoint라고 해봅시다.(또 $A \in \mathcal{A}$)

그러면 $\mathcal{L'}$의 정의에 따라 $A \cap B_i \in \mathcal{L_0}$임을 활용해보면$\bigcup_{i=1}^{\infty} (A \cap B_i) \in \mathcal{L_0}$겠죠? $\lambda$ system의 세 번째 정의에서 $B_i$가 pairwise disjoint하기 때문에 위 문장이 성립합니다.

 

즉, $\bigcup_{i=1}^{\infty} (A \cap B_i) = A \cap \bigcup_{i=1}^{\infty} B_i$이므로 $A \cap \bigcup_{i=1}^{\infty} B_i$도 $\mathcal{L_0}$에 속하게 됩니다.

그러므로 $\mathcal{L'}$의 정의에 따라 $$\bigcup_{i=1}^{\infty} B_i \in \mathcal{L'}$$입니다.

 

즉, $\mathcal{L'}$은 $\lambda$ system인데요. 정의에 따라 $\mathcal{L'} \subset \mathcal{L_0}$겠죠? $\mathcal{L_0}$ 내에서 택하여 $\mathcal{L'}$을 만들었으니까요. 그런데 다시 $\mathcal{L_0}$는 minimal, 즉, 가장 작은 $\lambda$ system입니다. 이로부터, $$\mathcal{L'} = \mathcal{L_0}$$임을 알 수 있겠습니다.

 

step2 Show that $\mathcal{L_0}$ is closed under intersection

이번엔 $\mathcal{L''} = \{B \in \mathcal{L_0} : B \cap A \in \mathcal{L_0}, \forall A \in \mathcal{L_0}\}$ 을 정의합시다.

그렇다면 $\mathcal{L_0}$에  $\mathcal{A}$가 속하기 때문에, $\mathcal{L'}$의 정의에 따라 $\mathcal{L'} = \mathcal{L_0} \subset \mathcal{L''}$이 성립합니다.

 

이때, $\mathcal{L''}$은 $\mathcal{L_0}$에서 택하여 정의했기 때문에 $\mathcal{L''} \subset \mathcal{L'}$입니다.

 

즉, $\mathcal{L''} = \mathcal{L_0}$이고, $\mathcal{L_0}$ 는 closed under intersection이라고 할 수 있겠습니다.

 

step3 proof by using the fact that collection of sets that is both $\lambda$ and $\pi$ system is $\sigma$-field

우리는 정의를 통해 $\lambda$ system이고 $\pi$ system 이면 $\sigma$-field임을 알 수 있습니다.

즉 $\mathcal{L_0}$가 $\sigma$-field임을 알 수 있죠.

 

그런데 모든  $\sigma$-field는 $\lambda$ system이므로 가장 작은 $\lambda$ system이 $\pi$ system이기까지 하니 $\mathcal{L_0}$(smallest)는 $\sigma (\mathcal{A})$(smallest)가 됩니다.

 

즉, $\sigma(\mathcal{A}) \subseteq \mathcal{L}$입니다.

그림으로 표현해 보면 아래와 같습니다.

결론

$\sigma(\mathcal{A}) \subseteq \mathcal{L}$라는 좋은 결과를 얻어냈는데 이는 measure의 uniquness를 증명할 때 쓰입니다!

사실 $\sigma(\mathcal{A})$의 의미를 글을 쓰는 중간까지도 잘못 알고 있었어서 고생을 좀 했습니다.

 

다음 포스팅은 uniqueness를 증명해 보겠습니다. 그런데 이제는 시간이 좀 부족해서 나중에나 올릴 것 같네요!

그럼 다음에 뵙겠습니다ㅏㅏ